Answer:
6 11/32
Step-by-step explanation:
The two cans of broth used in the recipe total 2·14.5 oz = 29 oz. The number of people John wants to serve is 28/4 = 7 times the number served by one recipe. So, the amount of broth needed is ...
(29 oz)·7 = 203 oz
Each quart container of broth holds 32 oz, so John will need ...
(203 oz)/(32 oz/container) = 6 11/32 containers
John needs 6 11/32 containers of broth.
_____
If John must buy the broth, he must buy 7 containers. he will have 21/32 of a container left over after making his soup.
What is an equation of a line that passes through the points (1, 4) and (2, 9) in
5)
point-slope form? Analyze the steps and fill in the blanks. (4pts)
First, use the two given points to find the slope.
m=
Уг-У1
Use the slope and one point to write an equation of the line in
point-slope form.
Point-slope form of a linear equation.
Substitute
The equation of line in point-slope form of the line, is y = 5x - 1.
What is point - slope form of the line?
The general form of a linear equation is y - y1 = m(x - x1).
It draws attention to the line's slope and one of the line's points (that is not the y-intercept).
Given:
The line passes through the points (1, 4) and (2, 9).
We have to find the equation of line in point - slope form.
First to find the slope from the given points.
Since,
\(m = \frac{y_2-y_1}{x_2-x_1}\)
Here, \((x_1, y_1) = (1, 4), (x_2, y_2) = (2, 9)\)
So, \(m = \frac{9-4}{2-1} = 5\)
Now consider, the point slope form of the line
\(y-y_1=m(x-x_1)\)
Plug \(m = 5, (x_1, y_1) = (1, 4)\)
⇒
\(y - 4=5(x-1)\\y-4=5x-5\\y=5x-5+4\\y=5x-1\)
Hence, the equation of line in point-slope form of the line, is y = 5x - 1.
To know more about point - slope form of the line, click on the link
https://brainly.com/question/24907633
#SPJ1
what is the missing number? 28, 29, 57, 86, __, 229
A company produces a special new type of TV. The company has fixed costs of $493,000, and it costs $1200 to produce each TV. The company projects that if it charges a price of $2200 for theTV, it will be able to sell 700 TVs. If the company wants to sell 750 TVs, however, it must lower the price to $1900. Assume a linear demand. What are the company's profits if marginal profit is $0? The profit will $enter your response here.
To find the company's profits, we need to calculate the total revenue and total cost. Since marginal profit is $0, the total revenue and total cost will be equal.
To calculate the total revenue, we multiply the price per TV by the number of TVs sold. At a price of $2200, the company sells 700 TVs, so the total revenue is 2200 * 700 = $1,540,000. At a price of $1900, the company sells 750 TVs, so the total revenue is 1900 * 750 = $1,425,000.
The total cost consists of fixed costs and variable costs. The variable cost is the cost to produce each TV multiplied by the number of TVs sold. The fixed costs are constant regardless of the number of TVs sold. The variable cost is $1200 per TV. At a price of $2200, the variable cost for 700 TVs is 1200 * 700 = $840,000. At a price of $1900, the variable cost for 750 TVs is 1200 * 750 = $900,000.
Therefore, the total cost at a price of $2200 is 493,000 + 840,000 = $1,333,000, and the total cost at a price of $1900 is 493,000 + 900,000 = $1,393,000.
Since marginal profit is $0, the total revenue is equal to the total cost. Thus, the company's profits are $1,540,000 - $1,333,000 = $207,000 at a price of $2200, and $1,425,000 - $1,393,000 = $32,000 at a price of $1900.
To learn more about Linear demand
brainly.com/question/29213712
#SPJ11
Fill in the table using this function rule.
y = 2x + 5
Answer:
The answer is
[ X ][ Y ]
[ 1 ][ 7 ]
[ 2 ][ 9 ]
[ 5 ][ 15 ]
[ 7 ][ 19 ]
Brainiest please and lot of likes:)
This is due today pls help! :)
Answer:
D
Step-by-step explanation:
8.4 minus 3 equals 5.4
10¹⁰ minus 10⁷ is 10³
Answer:
The correct answer is 2.8 x 10^3
Step-by-step explanation:
I Hope this helps.
answer question a and b please
HELP ASAP!
6.3 x 15 divided by 3 + 6.8 - 28.09
I NEED TO SOMEHOW GET 46.25 BUT IT DOESNT LEAD UP TO THAT PLEASE HELP
The required value of the expression is 10.21.
What is BODMAS?BODMAS is an acronym that stands for Brackets, Orders (exponents), Division, Multiplication, Addition, and Subtraction. It is a mathematical rule used to determine the order of operations to solve mathematical expressions or equations.
The order of operations is essential to ensure that everyone obtains the same answer when solving mathematical problems. Without following a set order, different people could interpret the same expression in different ways, which can lead to incorrect answers.
The BODMAS rule tells us to perform calculations in the following order:
\(6.3 \times 15 \div 3 + 6.8 - 28.09 \\\\= (6.3 \times 15) \div 3 + 6.8 - 28.09 \\\\= 94.5 \div 3 + 6.8 - 28.09 \\\\= 31.5 + 6.8 - 28.09 \\\\= 10.21 \end{aligned}\)
Therefore, the value of the expression is 10.21.
To know more about BODMAS visit:
brainly.com/question/29756765
#SPJ1
I'm sorry, but it is not possible to get a value of 46.25 by manipulating the given expression using standard arithmetic operations (+, -, ×, ÷). The expression evaluates to 10.21, which is a completely different value.
If you need to get a result of 46.25, you would need to modify the expression by adding or subtracting terms. For example, one possible expression that evaluates to 46.25 is:
(6.3 × 15 + 3) ÷ 3 + 6.8 - 28.09
This expression first multiplies 6.3 by 15 and adds 3 to obtain 96. Then it divides 96 by 3 to get 32. Finally, it adds 6.8 and subtracts 28.09 to obtain the final result of 46.25.
However, it is important to note that this modified expression is not equivalent to the original expression and involves additional terms.
below the paraboloid z = 18 − 2x2 − 2y2 and above the xy-plane
Answer:
y
2
=−
2
z
+7
Steps for Solving Linear Equation
z=18−2×2−2y2
Multiply 2 and 2 to get 4.
z=18−4−2y
2
Subtract 4 from 18 to get 14.
z=14−2y
2
Swap sides so that all variable terms are on the left hand side.
14−2y
2
=z
Subtract 14 from both sides.
−2y
2
=z−14
Divide both sides by −2.
−2
−2y
2
=
−2
z−14
Dividing by −2 undoes the multiplication by −2.
y
2
=
−2
z−14
Divide z−14 by −2.
y
2
=−
2
z
+7
Step-by-step explanation:
the given equation defines a paraboloid that lies below the plane z=0. Specifically, it is situated above the xy-plane, which means that the z-values of all points on the surface are greater than or equal to zero.
we can break down the equation z=18-2x^2-2y^2. This equation represents a paraboloid with its vertex at (0,0,18) and axis of symmetry along the z-axis. The first term 18 is the z-coordinate of the vertex and the last two terms -2x^2 and -2y^2 determine the shape of the paraboloid.
Since the coefficient of x^2 and y^2 terms are negative, the paraboloid is downward facing and opens along the negative z-axis. Therefore, all points on the paraboloid have z-values less than 18. Additionally, since the paraboloid is situated above the xy-plane, its z-values are greater than or equal to zero.
the paraboloid defined by the equation z=18-2x^2-2y^2 is situated below the plane z=0 and above the xy-plane. Its vertex is at (0,0,18) and it opens along the negative z-axis.
To know more about paraboloid, visit:
https://brainly.com/question/30925041
#SPJ11
Evaluate 7+q(r−8)2 when q = 3 and r = 10.
Answer:
The solution of the expression is 19.Step-by-step explanation:
Given:
q = 3r = 10Work:
=> 7 + q(r − 8)2=> 7 + 3(10 - 8)2=> 7 + 6(10 - 8)=> 7 + 6(2)=> 7 + 12=> 19Hence, the solution of the expression is 19.
Hoped this helped.
\(BrainiacUser1357\)
Answer:
19
Step-by-step explanation:
7 + q (r − 8) 2
7 + q (10 - 8) 2
7 + q * 2 * 2
7 + 3 * 2 * 2
7 + 6 * 2
7 + 12
19
L 4.6.3 Test (CST): Linear Equations
me.
OA. y+4= -3(x-3)
OB. y-4=-3(x+3)
OC. y-4=3(x+3)
OD. y+4=3(x-3)
(3,-4)
The correct option is OA. y+4= -3(x-3). L 4.6.3 Test (CST): Linear Equations Solution: We are given that a line passes through (3,-4) and has a slope of -3.
We will use point slope form of line to obtain the equation of liney - y1 = m(x - x1).
Plugging in the values, we get,y - (-4) = -3(x - 3).
Simplifying the above expression, we get y + 4 = -3x + 9y = -3x + 9 - 4y = -3x + 5y = -3x + 5.
This equation is in slope intercept form of line where slope is -3 and y-intercept is 5.The above equation is not matching with any of the options given.
Let's try to put the equation in standard form of line,ax + by = c=> 3x + y = 5
Multiplying all the terms by -1,-3x - y = -5
We observe that option (A) satisfies the above equation of line, therefore correct option is OA. y+4= -3(x-3).
Thus, the correct option is OA. y+4= -3(x-3).
For more question on equation
https://brainly.com/question/17145398
#SPJ8
Which table represents y as a function of x?
PLEASE HELP??
OR BRAINLIEST:((??
Which set of angle measures can be used to form a triangle ? A : 40 , 60 , 85 ,, B : 45 , 60 , 60 , C : 30 , 60 , 90 , D : 45 , 90 , 90
Answer:
D)45,90 90 this is the answer
Answer:
C) 30, 60, 90
Step-by-step explanation:
all three angles must add up to 180
two fair dice are tossed, and the up face on each die is recorded. a. list the 36 sample points contained in the sample space. b. assign probabilities to the sample points. c. find the probability of observing each of the following events: a: 5a 3 appears on each of the two dice.6 b: 5the sum of the numbers is even.6 c: 5the sum of the numbers is equal to 7.6 d: 5a 5 appears on at least one of the dice.6 e: 5the sum of the numbers is 10 or more.6
a. The probability of getting 3 on both dice is 1/36.
b. The probability of each sample point is 1/36.
c. The probability of getting a sum of 7 is 6/36 or 1/6.
d. The probability of getting at least one 5 is 11/36.
e. The probability of getting a sum of 10 or more is 6/36 or 1/6.
a. The sample space for tossing two fair dice can be listed as follows:
(1,1), (1,2), (1,3), (1,4), (1,5), (1,6)
(2,1), (2,2), (2,3), (2,4), (2,5), (2,6)
(3,1), (3,2), (3,3), (3,4), (3,5), (3,6)
(4,1), (4,2), (4,3), (4,4), (4,5), (4,6)
(5,1), (5,2), (5,3), (5,4), (5,5), (5,6)
(6,1), (6,2), (6,3), (6,4), (6,5), (6,6)
b. Since the dice are fair, each outcome is equally likely.
Therefore, the probability of each sample point is 1/36.
a. The event of getting 3 on both dice can only happen in one way, that is, (3,3).
Therefore, the probability of getting 3 on both dice is 1/36.
b. For the sum of the numbers to be even, either both dice must be odd or both must be even.
There are 3 odd numbers (1, 3, 5) and 3 even numbers (2, 4, 6) on a die. Thus, there are 3 x 3 = 9 ways to get an even sum.
The probability of getting an even sum is therefore 9/36 or 1/4.
c. To get a sum of 7, the following combinations are possible: (1,6), (2,5), (3,4), (4,3), (5,2), and (6,1).
Therefore, there are 6 ways to get a sum of 7.
Thus, the probability of getting a sum of 7 is 6/36 or 1/6.
d. To get at least one 5, we can either get a 5 on the first die or the second die or both.
There are two ways to get a 5 on one die and one way to get a 5 on both dice.
Thus, the probability of getting at least one 5 is 11/36.
e. There are several ways to get a sum of 10 or more: (4,6), (5,5), (5,6), (6,4), (6,5), and (6,6).
Therefore, there are 6 ways to get a sum of 10 or more.
Thus, the probability of getting a sum of 10 or more is 6/36 or 1/6.
For similar question on probability.
https://brainly.com/question/29863918
#SPJ11
Sweats--Coach Tis looking for new sweatpants at Tarjay. He sees that Mikey brand sweatpants are $49.99 each but they are 15% off with a teacher discount. What final price does Coach pay for one new pair? (You should round to the nearest cent) "The final price Coach pays is..."
Answer:
your english is very bad
Step-by-step explanation:
Determine the equation of the line that passes through the point (6,-32) and is parallel to the line y=-5x-3
PLEASE GIVE BRAINLIEST!
I hope this helped :)
Answer:
y = -5x - 2
Step-by-step explanation:
Two lines are parallel if they have the same slope. The given line has a slope of -5, so the line we want to find will also have a slope of -5.
Using the point-slope formula:
y - y1 = m(x - x1)
where m is the slope and (x1, y1) is the given point, we have:
y - (-32) = -5(x - 6)
Simplifying:y + 32 = -5x + 30y = -5x - 2
Therefore, the equation of the line that passes through the point
(6, -32) and is parallel to the line y = -5x - 3 is y = -5x - 2.
Answer:
Step-by-step explanation:
when trying to find the parallel line the slope is the same.
equation: y = mx + bso m = -5then, you need to find out what b is. when finding b, you already have the equation y = -5x + bwith that, all you need to do is plug in the coordinates.so that would make it -32 = -5(6) + bthen solve:-32 = -36 + b
+36 +36
4 = b
and now you have the equation of the parallel line which is y = -5x + 4
chegg amazon ships several parcels each day to multiplelocations. the dispatch centers have conveyor belts where the parcel boxes are placed. in one such center, there are a total of n boxesnumbered 0, 1,., (n - 1), where the capacity of the ith box is denoted by capacity[i]. a box numbered x, can contain a box numbered y, if capacity[x] is divisible by capacity[y]
The correct answer is a box numbered x can contain a box numbered y if capacity[x] is divisible by capacity[y]. The containment relationships can be determined based on this criterion.
In the given scenario, Amazon ships several parcels each day to multiple locations, and the dispatch center has conveyor belts where the parcel boxes are placed. The boxes in the center are numbered from 0 to (n - 1), and the capacity of each box is denoted by capacity[i]. A box numbered x can contain a box numbered y if the capacity[x] is divisible by capacity[y].
To better understand the situation, let's consider an example. Suppose we have 5 boxes numbered from 0 to 4, and their respective capacities are as follows:
capacity[0] = 10
capacity[1] = 20
capacity[2] = 5
capacity[3] = 2
capacity[4] = 1
Based on the given conditions, the box numbered 0 can contain any other box since any number is divisible by 1. The box numbered 1 can contain boxes 0 and 2, as 20 is divisible by 10 and 5. The box numbered 2 can contain only box 4 since 5 is divisible by 1. The box numbered 3 can contain only box 4 as 2 is divisible by 1. Finally, the box numbered 4 cannot contain any other box since it has the smallest capacity.
So, in this case, the possible containment relationships are as follows:
0 can contain: 1, 2, 3, 4
1 can contain: 0, 2
2 can contain: 4
3 can contain: 4
4 cannot contain any other box
Learn more about vector here:
https://brainly.com/question/28028700
#SPJ11
i need help please branliest to who gets it right
The kite is about 16.5 yards above the edge of the pond.
How to find the height of the kite?You are flying a dragon kite. It's connected to 36 yards string. The kite is directly above the end of the pond. The edge of the pond is 32 yards where the kite is tied to the ground.
Therefore, the height of the kite above the ground can be calculated as follows:
This situation form a right angle triangle. Hence, using Pythagoras's theorem,
Therefore,
h² = 36² - 32²
h² = 1296 - 1024
h = √272
h = 16.4924225025
Hence, the height of the kite is 16.5 yards.
learn more on right triangle here: brainly.com/question/28975129
#SPJ1
can someone pls help Write the following equation as a sentence: w = (15)(165)
Answer:
width equals fifteen times one hundred sixty five
is that right or is that ok or good enough?? sorry if i am wrong....
Answer:
W = 2475
Step-by-step explanation:
I am not sure if you want the answer or not but I hope this helps
Give the following FAs over the alphabet >=(0,1,2} (30 pt) a-) A DFA for { strings in which the number of even digits is odd} b-) A DFA for { strings, when interpreted as a base-3 number, are even numbers } c-) A DFA for { strings, when interpreted as a base-3 number, are even numbers having odd number of even digits } d-) A DFA for { strings, when interpreted as a base-3 number, are not an even number} e-) An -NFA for {]o, when interpreted as a base-3 number, is an even number}
The accepting state will be state 0 since it represents even numbers in base-3. The NFA can transition to both states at the beginning, but it only needs to be in the accepting state at the end to accept an even number.
a) DFA for {strings in which the number of even digits is odd}:
The DFA will have two states: an even state and an odd state. The initial state will be the even state. When the DFA reads an odd digit (1 or 3), it will transition to the other state. When it reads an even digit (0 or 2), it will stay in the same state. The accepting state will be the odd state. This DFA will accept strings in which the number of even digits is odd.
b) DFA for {strings, when interpreted as a base-3 number, are even numbers}:
The DFA will have three states: state 0, state 1, and state 2. The initial state will be state 0. When the DFA reads a digit, it will transition to the state corresponding to that digit. For example, if it reads a 0, it will transition to state 0; if it reads a 1, it will transition to state 1, and if it reads a 2, it will transition to state 2. The accepting state will be state 0 since it represents even numbers in base-3.
c) DFA for {strings, when interpreted as a base-3 number, are even numbers having an odd number of even digits}:
This DFA will have three states: state 0, state 1, and state 2. The initial state will be state 0. When the DFA reads a digit, it will transition to the state corresponding to that digit. Similar to the previous DFA, the accepting state will be state 0. However, we need to keep track of the number of even digits. To achieve this, we can introduce a counter. Every time the DFA transitions to state 2, representing an even digit, the counter will be incremented. If the counter becomes odd, the DFA will transition to state 1. In state 1, it will accept only odd numbers of even digits.
d) DFA for {strings, when interpreted as a base-3 number, are not an even number}:
This DFA will have three states: state 0, state 1, and state 2. The initial state will be state 0. When the DFA reads a digit, it will transition to the state corresponding to that digit. The accepting state will be state 1 and state 2 since these states represent odd numbers in base-3.
e) NFA for {strings, when interpreted as a base-3 number, is an even number}:
The NFA will have two states: state 0 and state 1. The initial state will be state 0. When the NFA reads an odd digit (1 or 2), it will transition to both state 0 and state 1. When it reads an even digit (0), it will stay in the same state. The accepting state will be state 0 since it represents even numbers in base-3. The NFA can transition to both states at the beginning, but it only needs to be in the accepting state at the end to accept an even number.
Learn more about transition here
https://brainly.com/question/17145924
#SPJ11
Given are the numbers of 30 test scores in a math class. 56, 69, 76, 84, 102, 69, 78, 92, 79, 80, 66, 78, 90, 99, 45, 76, 77, 80, 57, 57, 85, 88, 55, 86, 99, 97, 87, 75, 65, 70 a. Create a histogram to represent this data. (3points) b. Find the mean, median, mode and standard deviation. (3 points) c. Determine the five-number summary and draw a boxplot. (4 points)
a) Histogram is attached. b) Mean is 75.8, median is 78.5, mode is 57, Standard Deviation is 15.56. c) Five- line summary is Minimum: 45, Q1: 66, Median: 78.5, Q3: 87, Maximum: 102.
a. To create a histogram for the given data, we need to divide the range of scores into intervals (bins) and count the frequency of scores falling within each interval.
Scores Frequency
40-50 |
50-60 |||
60-70 ||||||||
70-80 |||||
80-90 |||||
90-100 ||||||
100-110 |
b) Calculate the mean:
To find the mean, we sum up all the test scores and divide by the total number of scores.
Mean = (56 + 69 + 76 + 84 + 102 + 69 + 78 + 92 + 79 + 80 + 66 + 78 + 90 + 99 + 45 + 76 + 77 + 80 + 57 + 57 + 85 + 88 + 55 + 86 + 99 + 97 + 87 + 75 + 65 + 70) / 30
Mean = 2274 / 30
Mean = 75.8
Therefore, the mean of the test scores is 75.8.
Calculate the median:
To find the median, we need to arrange the scores in ascending order and then determine the middle value.
Arranging the scores in ascending order: 45, 55, 56, 57, 57, 65, 66, 69, 69, 70, 75, 76, 76, 77, 78, 78, 79, 80, 80, 84, 85, 86, 87, 88, 90, 92, 97, 99, 99, 102
Since we have 30 scores, the median will be the average of the 15th and 16th values, which are 78 and 79.
Median = (78 + 79) / 2
Median = 78.5
Therefore, the median of the test scores is 78.5.
Calculate the mode:
The mode is the value(s) that appear(s) most frequently in the data set.
The mode of the given test scores is 57 because it appears twice, which is more frequently than any other value.
Therefore, the mode of the test scores is 57.
Calculate the standard deviation:
The standard deviation measures the spread of the data around the mean.
To calculate the standard deviation, we can use the following formula:
Standard Deviation = \(\sqrt{(xi-mean)^{2}/n }\)
Where xi represents the sum, xi is each individual score, mean is the mean we calculated earlier, and n is the total number of scores.
Using this formula, we can calculate the standard deviation as follows:
Standard Deviation = \(\sqrt{((56-75.8)^{2}+(69-75.8)^{2}........+(70-75.8)^{2})/30}\)
After calculating all the differences and summing them up, we divide by 30 and take the square root.
Standard Deviation ≈ 15.56
Therefore, the standard deviation of the test scores is approximately 15.56.
c) Calculate the five-number summary and draw a boxplot:
The five-number summary consists of the minimum, first quartile (Q1), median (Q2), third quartile (Q3), and maximum values of the data set.
Minimum: 45
Q1: 66
Median: 78.5
Q3: 87
Maximum: 102
To draw a boxplot, we use these five values. The box represents the interquartile range (Q3 - Q1), with a line inside representing the median. The whiskers extend to the minimum and maximum values.
To learn more about Histogram here:
https://brainly.com/question/16819077
#SPJ4
Solve the equation the square root of the quantity x plus 4 minus 3 equals for the variable. Show each step of your solution process.
Answer:
√(x + 4) - 3 = 0 is x = 5.
Step-by-step explanation:
To solve the equation √(x + 4) - 3 = 0, we can follow these steps:
Step 1: Add 3 to both sides of the equation to isolate the square root term:
√(x + 4) = 3
Step 2: Square both sides of the equation to eliminate the square root:
(√(x + 4))^2 = 3^2
Simplifying:
x + 4 = 9
Step 3: Subtract 4 from both sides of the equation to isolate the variable x:
x = 9 - 4
Simplifying:
x = 5
Thus, the solution to the equation √(x + 4) - 3 = 0 is x = 5.
A large nationwide poll recently showed an unemployment rate of 9% in the US. The mayor of a local town wonders if this national result holds true for her town, so she plans on taking a sample of her residents to see if the unemployment rate is significantly different than 9% in her town.
Let p represent the unemployment rate in her town. Here are the hypothesis she'll use:
H0 : p=0.09
H1 : p≠0.09
Under which of the following conditions would the mayor commit a Type I error?
a. She concludes the town's unemployment rate is not 9% when it actually is.
b. She concludes the town's unemployment rate is not 9% when it actually is not.
c. She concludes the town's unemployment rate is 9% when it actually is.
d. She concludes the town's unemployment rate is 9% when it actually is not.
Type I error is formed when she concludes the town's unemployment rate is not 9% when it actually is, which is an option (a).
Given that:
A large nationwide poll recently showed an unemployment rate of 9% in the US.
The mayor of a local town wonders if this national result holds true for her town, so she plans on taking a sample of her residents to see if the unemployment rate is significantly different than 9% in her town.
Here the null hypothesis is taken as that the unemployment rate is 9%, whereas the alternative hypothesis is that it is not 9%.
Type I error is the error formed when the null hypothesis is rejected, but the hypothesis is actually true.
So, here the conclusion is that the unemployment rate is not 9%, but it actually is 9%.
Hence, the correct option is (a).
Learn more about Errors here :
https://brainly.com/question/34299120
#SPJ12
What is measure of angle 4 measure of angle 5 measure of angle 6? a triangle has angles 1, 2, 3. the exterior angle to angle 1 is 4, to angle 2 is 5, to angle 3 is 6. 180° 270° 300° 360°
Measure of angle 4 + measure of angle 5 + measure of angle 6 = 360°
The correct answer is an option (d)
Here, the exterior angle to angle 1 is 4.
We can observe that angle 1 and angle 4 are linear angles.
So, ∠1 + ∠4 = 180° .......(1)
Similarly angle 5 is an exterior angle to 5,
⇒ ∠2 + ∠5 = 180° ..............(2)
And the exterior angle to angle 3 is 6.
⇒ ∠3 + ∠5 = 180° ...........(3)
Consider the sum Measure of angle 4 + measure of angle 5 + measure of angle 6:
= m∠4 + m∠5 + m∠6
= 180° - m∠1 + 180° - m∠2 + 180° - m∠3 .....(from (1), (2) and (3))
= 540° - ( m∠1 + m∠2 + m∠3)
= 540° - 180° ........(sum of all angles of triangle is 180°)
= 360°
Therefore, the required sum = 360°
The correct answer is an option (d)
Learn more about triangle here:
https://brainly.com/question/22469440
#SPJ4
The complete question is:
What is Measure of angle 4 + measure of angle 5 + measure of angle 6?A triangle has angles 1, 2, 3. The exterior angle to angle 1 is 4, to angle 2 is 5, to angle 3 is 6.
a) 180°
b) 270°
c) 300°
d) 360°
BRAINIEST, THANKS, AND 5 STAR RATING what is the equation of the line in slope intercept form?
Answer:
y = -300x
Step-by-step explanation:
The y-intercept passes through 0, in this case, we don't have to include the y intercept to the equation. The slope is -300, or vertical/horizontal (rise/run, the formula for finding slope <<) We just include the slope if there is no y intercept. So, y = -300x is the equation!
Monica rode her bike 3 1/4 miles on monday.she rode 2 2/4 miles on tuesday how much farther did monica ride on minday than tuesday
Answer:
3/4 miles
Step-by-step explanation:
3 1/4 as a mixed fraction => 13/4
2 2/4 as a mixed fraction => 10/4
13/4 - 10/4 = 3/4
She rode 3/4 miles more on Monday than Tuesday.
Answer:
Step-by-step explanation:
nbhbhbnnmnmnjn
The total length of a road trip was 13.2 hours. If highway signs are posted every 0.06 hours, including one at the end of the road trip, how many highway signs will there be on the road trip?
There will be 221 highway signs on the road trip.
What is Total Length?
Whole Length (TL) is the measurement taken in a straight line, with the fish lying on its side, from the tip of the snout to the end of the tail (caudal fin).
The total length of the road trip is 13.2 hours.
Highway signs are posted every 0.06 hours. To find out how many highway signs are on the road trip, we can divide the total length of the road trip by the interval between each sign and then add one more sign for the end of the road trip:
(number of signs) = (total length of road trip) / (interval between signs) + 1
Substituting the values, we get:
(number of signs) = 13.2 / 0.06 + 1
(number of signs) = 220 + 1
(number of signs) = 221
Therefore, there will be 221 highway signs on the road trip.
To know more about Total Length visit,
https://brainly.com/question/26319327
#SPJ1
Find the Next 3 Letters in J F M A M J J A
What are the next 3 letters in the sequence J F M A M J J A?
The next three letters in the sequence J F M A M J J A are S, O, N.
To find the next three letters in the sequence J F M A M J J A, we need to identify the pattern or rule that governs the sequence. In this case, the sequence follows the pattern of the first letter of each month in the year.
The sequence starts with 'J' for January, followed by 'F' for February, 'M' for March, 'A' for April, 'M' for May, 'J' for June, 'J' for July, and 'A' for August. The pattern repeats itself every 12 months.
Therefore, the next three letters in the sequence would be 'S' for September, 'O' for October, and 'N' for November.
Learn more:About sequence here:
https://brainly.com/question/30262438
#SPJ11
The next three letters in the sequence "J F M A M J J A" are "S O N", indicating the months of September, October, and November.
The given sequence "J F M A M J J A" represents the first letters of the months in a year, starting from January (J) and ending with August (A). To find the next three letters in the sequence, we need to continue the pattern by considering the remaining months.
The next month after August is September, so the next letter in the sequence is "S". After September comes October, represented by the letter "O". Finally, the month following October is November, which can be represented by the letter "N".
Therefore, the next three letters in the sequence "J F M A M J J A" are "S O N", indicating the months of September, October, and November.
It is important to note that the given sequence follows the pattern of the months in the Gregorian calendar. However, different cultures and calendars may have different sequences or names for the months.
Learn more about: sequence
https://brainly.com/question/30262438
#SPJ11
what is this answer???
Answer:
<AEB = 51.2
Step-by-step explanation:
Remark
A straight line has a degree value of 180. There are 3 angles. One of them is 90. The other two must add to 90
Equation
2.7x + 8 + 3.8x - 22 = 90 combine like terms on the left.
6.5x - 14 = 90 add 14 to both sides
6.5x = 90 + 14
6.5x = 104 Divide by 6.5
x = 104/6.5
x = 16
<AEB = 2.7*16 + 8
<AEB = 51.2
Scientists say we should sleep 3/8 of a day.How about many hours is this
Answer:
9 hours
Step-by-step explanation:
Answer:
\(\huge\color{skyblue}\boxed{\colorbox{black}{Answer ☘}}\)
number of hours in a day = 24
according to scientists ,
we should sleep number of hours = 3/8
therefore ,
\( \frac{3}{8} \: of \: 24 \: hours = \frac{3}{8} \times 24 \\ = > 9 \: hours\)
hope helpful~
A club has 15 members and needs to choose 2 members to be co-presidents. In how many ways can the club choose its co-presidents?
Answer:
15 choose 2 = 105
\(=\frac{15!}{2!\left(15-2\right)!}\)
Step-by-step explanation: